Determining the market risk premium


Assignment:

Currently in the market, the risk-free rate is 6% and the market risk premium [E(RM) - RF] is 5%. Based on this information, which of the following statements is most correct? Assume that CAPM is correct.

A If a stock has a negative beta, its required return must also be negative.

b. If a stock's beta doubles, its required return must also double.

c. A portfolio with a beta=1.0 has a required return of 11%.

d. Answers a. and c. are both correct.

e. Answers b. and c. are both correct.

Solution Preview :

Prepared by a verified Expert
Portfolio Management: Determining the market risk premium
Reference No:- TGS02015435

Now Priced at $20 (50% Discount)

Recommended (93%)

Rated (4.5/5)